Dual rotor axial flux motor design

rhitee05 said:
Hot off the presses:

View attachment 1


These data are for the same model as before, except another tweak to airgap/magnet thickness.

I'm running a set of simulations overnight to check the 6 mm wide, 28 mm long configuration for 3 different magnet thicknesses.
Thats cool its neet to see how torque goes down as you up current past I certian point. I proved this with my X5 last summer on my little dyno.
But I dont understand the torque constant vs phas amps...? Why does it go down as you add phase amps?
 
rhitee05 said:
What airgap are you comfortable with? I took it back down to 0.5 mm (on each end) in the current model, but it could be increased again to 0.75 or 1 mm (with thicker magnets) if you want some larger tolerances.
What magnet strength are you modelling for? I would prefer to use N48 with 0.75mm airgap and make the final adjustment by magnet thickness.
 
rhitee05 said:
I also meant to ask, are the retention pins you've placed in the end of each core going to be steel? I have them modeled as air right now, but if they're going to be steel that might make a (slight) change to the characteristics.
Yes, they'll probably be steel. I'm relying more on them more structurally now, so I've decided to use M1.6mm A2 stainless steel torx screws. Their major dia. measures at 1.45mm, so they will fit in a 1.5mm hole. Hope that's not going to be too detrimental....? The path on either side of the hole will be about half the neck width. We could raise the hole slightly (nearer to the head surface), if that would help?
 
Miles said:
What magnet strength are you modelling for? I would prefer to use N48 with 0.75mm airgap and make the final adjustment by magnet thickness.

The model is still using N30EH magnets. I can change to N48, make the airgap 0.75 mm, and then adjust thickness as needed. Narrowing down the configuration options... :)

Miles said:
Yes, they'll probably be steel. I'm relying more on them more structurally now, so I've decided to use M1.6mm A2 stainless steel torx screws. Their major dia. measures at 1.45mm, so they will fit in a 1.5mm hole. Hope that's not going to be too detrimental....? The path on either side of the hole will be about half the neck width. We could raise the hole slightly (nearer to the head surface), if that would help?

Okay, if the screws will be stainless steel then I need to change the magnetic parameters back to air (SS is not magnetic). I can also tweak the holes to 1.5 mm. I suspect it's probably best just to leave the holes centered where they are. Later, I can try running a couple of simulations for stainless vs regular steel to see how much effect there is.
 
Arlo1 said:
Thats cool its neet to see how torque goes down as you up current past I certian point. I proved this with my X5 last summer on my little dyno.
But I dont understand the torque constant vs phas amps...? Why does it go down as you add phase amps?

Well, if the torque output levels off or decreases for increasing current, then the torque constant will naturally decrease (torque/current).

The physical cause for this is that the steel is becoming heavily saturated, so large changes in current only translate to small changes in the fields and thus small changes in torque.
 
rhitee05 said:
The model is still using N30EH magnets. I can change to N48, make the airgap 0.75 mm, and then adjust thickness as needed. Narrowing down the configuration options... :)
Now we've increased the core height, the magnet thickness is starting to matter :)
 
New set of model runs: N30EH magnets, 0.5 mm airgap, 6 mm wide x 28 mm long cores, 2x3t coils, and 1.3 mm dia steel pins in core ends. Obviously several of these parameters will be changed, as discussed, but these results are VERY promising.

3 mm thick magnets:
Model4a Torque vs Current.png
Model4a BEMF.png

4 mm thick magnets:
Model4b Torque vs Current.png
Model4b BEMF.png

5 mm thick magnets:
Model4c Torque vs Current.png
View attachment 1

How does a peak torque of >6 N-m sound to you? :-D

I updated the model to reflect our most recent discussion: 0.75 mm airgap, N48 magnets, 1.5 mm dia stainless (modeled as air) pins. I just started a set of runs for 1.5, 3, 4, and 5 mm magnet thicknesses.
 
I calculate Rm to be 0.0136 Ohms

For Kt of 0.055 Nm/A that makes Km 0.47

So, specific Km is now 0.41 :D

Fill factor for the 2x3t coils is up to 0.95

It's starting to look as if it might be worth making one.... :)
 
Miles said:
Can we get some idea of the iron-losses? It would be good to know max. Eta.

I can try, but I haven't done that in FEMM before and I'm not sure how accurate it will be. I know that FEMM can't take into account the motion-induced eddy current losses, and the current-induced eddy losses it calculates will be assuming sinusoidal currents. Point is, it can be done, but I'm not sure you'd do much worse with a blindfold and dart board. :) I think it makes sense to save this for later, when we either have a final topology chosen or a handful of possibilities. There might be some other ways we could investigate these losses, I'd have to do some research.

Speaking of which, I've been thinking and trying to define a suite of parameter variations to run. I'm running the model now for 6mm wide cores and a variety of N48 magnet thicknesses. When those are done, I think I will try the same set with 5mm wide cores. Having been shown the light that high static flux density is not a bad thing, I want to re-visit some of the narrower cores. I might also run with 4mm cores. That will give us a reasonable idea of performance against the combination of those two parameters. I think that should give you some options to choose from, like the lighter and slightly more efficient motor that tops out not much past 4 N-m, or the larger and slightly less efficient motor than can make 6+ N-m if you drive it hard. Are there any other parameters that we would also like a suite of data against? We could run for various core lengths, but I don't think that would make much of a difference magnetically - it's mostly a factor in how much copper volume is available - so I'm inclined not to bother until possibly at the end.

I will be running a suite of simulations for back-iron thickness, but I saving that for later when we've narrowed it down to one or two options. For the time being, I've made the back iron very thick to remove it as a limiting factor in the motor. Later I'll run some models for reduced thicknesses and we can determine where it starts to cut into performance. I'm getting the impression that you have a specific total length target you want to hit, so this might be a place to run a set of models for a given total length for the trade-off between magnet and back-iron thickness.
 
rhitee05 said:
I can try, but I haven't done that in FEMM before and I'm not sure how accurate it will be. I know that FEMM can't take into account the motion-induced eddy current losses, and the current-induced eddy losses it calculates will be assuming sinusoidal currents. Point is, it can be done, but I'm not sure you'd do much worse with a blindfold and dart board. :) I think it makes sense to save this for later, when we either have a final topology chosen or a handful of possibilities. There might be some other ways we could investigate these losses, I'd have to do some research.
Ok. It's not that important WRT meeting the Challenge.
 
Biff did something to estimate iron losses in the 80100. I haven't checked how he did it, but it may work as inspiration.

http://endless-sphere.com/forums/viewtopic.php?p=302676#p302676
http://endless-sphere.com/forums/viewtopic.php?p=302816#p302816
http://endless-sphere.com/forums/viewtopic.php?p=491085#p491085
 
rhitee05 said:
I will be running a suite of simulations for back-iron thickness, but I saving that for later when we've narrowed it down to one or two options. For the time being, I've made the back iron very thick to remove it as a limiting factor in the motor. Later I'll run some models for reduced thicknesses and we can determine where it starts to cut into performance. I'm getting the impression that you have a specific total length target you want to hit, so this might be a place to run a set of models for a given total length for the trade-off between magnet and back-iron thickness.

I have an overall length of 56mm, which is set by the length of my gearbox.

The 28mm core height seems right - I'm happy with that.

That leaves probably a maximum of 6mm for magnet plus back-iron height (43.5 mm over back-irons). I'll see if I can gain some more, though.... :)
 
Miles said:
I have an overall length of 56mm, which is set by the length of my gearbox.

The 28mm core height seems right - I'm happy with that.

That leaves probably a maximum of 6mm for magnet plus back-iron height (43.5 mm over back-irons). I'll see if I can gain some more, though....

We can treat this as a tradeoff. For example, if we reduce the cores to 26 mm, that gains us 1mm on each side for additional magnet and/or iron. The tradeoff there should be a little extra loss (lower copper volume) for extra torque. We could run a few cases to get an idea of where the optimum is.

bearing said:
Biff did something to estimate iron losses in the 80100. I haven't checked how he did it, but it may work as inspiration.

Thanks, I'll take a look when I have some time!
 
rhitee05 said:
We can treat this as a tradeoff. For example, if we reduce the cores to 26 mm, that gains us 1mm on each side for additional magnet and/or iron. The tradeoff there should be a little extra loss (lower copper volume) for extra torque. We could run a few cases to get an idea of where the optimum is.
Ok. That makes sense.
 
Miles said:
Flux is 3D in the back-iron... Could we not increase the effective back-iron volume by reducing its inner radius?

Yes, that should work at least to some degree. I'm not sure if it would be a straight volume improvement (i.e. is 12 mm wide x 4 mm thick equivalent to 16 mm x 3 mm), but probably at least a fairly large percentage.
 
Miles said:
I have an overall length of 56mm, which is set by the length of my gearbox.

The 28mm core height seems right - I'm happy with that.

That leaves probably a maximum of 6mm for magnet plus back-iron height (43.5 mm over back-irons). I'll see if I can gain some more, though....

Trying to make sure I properly understand the length constraint. You said 43.5 mm total length, i.e. from the back edge of back iron on one rotor to back edge of back iron on the 2nd rotor? Using 28 mm long cores + 2*0.75 mm airgaps, that's 29.5 mm, which leaves 7 mm at each end for magnets + iron. Am I correct here? Going to 26 mm long cores would make it 8 mm each, and so forth. 24/26/28 seems like an appropriate range to explore here.
 
rhitee05 said:
Trying to make sure I properly understand the length constraint. You said 43.5 mm total length, i.e. from the back edge of back iron on one rotor to back edge of back iron on the 2nd rotor? Using 28 mm long cores + 2*0.75 mm airgaps, that's 29.5 mm, which leaves 7 mm at each end for magnets + iron. Am I correct here? Going to 26 mm long cores would make it 8 mm each, and so forth. 24/26/28 seems like an appropriate range to explore here.
:oops: Looks like I meant 41.5mm back edge to back edge of the back-irons. Sorry about that.

The only way I could get more would be to fix the back-iron to the top of the rotor (cantilevered out). The limit then would be 45mm.
 
Miles said:
The only way I could get more would be to fix the back-iron to the top of the rotor (cantilevered out). The limit then would be 45mm.

Or use a Halbach array :wink:
I'd stick with back iron though. A Halbach array would require some hard to find magnets.

Increasing the volume by decreasing the inner radius will only help a little, as the iron between the magnets will tend to saturate first, though satruating the back iron isn't all that bad as long as there isn't any aluminum right behind it.

I'm not sure what happens to the back iron at high currents. I suspect the flux will increase, so you need to avoid saturation at the maximum current. If the back iron saturates and the motor case is close to it, you could get some nasty eddy currents (drag).
 
fechter said:
Increasing the volume by decreasing the inner radius will only help a little, as the iron between the magnets will tend to saturate first, though satruating the back iron isn't all that bad as long as there isn't any aluminum right behind it.

That was my initial thinking as well, but after considering it some more I don't think this is as much of an issue.

At lower frequencies (where inductance is not an issue), electric currents will tend to spread over all the available copper, even if it's not the most direct path. If you have a PCB with ground plane, the return currents will tend to spread over the entire plane (more concentrated on the direct path between source and load, but widely distributed). This, incidentally, is where ground loops come from. The currents tend to be spread according to the resistances of the various paths.

Since we can make a good analogy to magnetic circuits, the same principle will hold true. As the iron directly behind the magnets begins to saturate, the flux will tend to spread out more and make use of all available iron even if the path length is a little longer. It might not be quite the same effectiveness as if you put the same additional volume of iron directly behind, but I think it would still be effective enough to be worthwhile. It's impossible to simulate this 3D flux pattern in FEMM, though.
 
MFEA is only so good. I don't really trust it fully but is a good tool for comparing alternate configurations.

I have a gaussmeter that I trust much more. If you take a pair of magnets and sandwich them between two pieces of iron to create a closed flux path, you can measure the flux on the back side. Maybe in this case, two pairs of magnets stacked and stuck together like having the two rotors stuck to each other without the stator might be close to operating conditions.

Based on what I've seen in other motors, I'd guess you want at least 4-5mm of back iron.

Even without a fancy gaussmeter, you can just take a paper clip and see how much attraction there is on the back side. If the iron is saturated, the clip is strongly attracted. On my Zappy motor, I know the back iron is saturated and it's close to 4mm. I think the magnets are about 5mm but I have no idea what grade they are.

Even if the back iron is saturated, the eddy currents will be greatly diminshed by leaving a good sized air gap between the iron and any stationary housing metal. Using stainless steel or titanium instead of aluminum makes a huge difference too.
 
Back
Top